Cardio Study Guide

Lakukan tugas rumah & ujian kamu dengan baik sekarang menggunakan Quizwiz!

A patient's CO is 5.5L/min and her heart rate is 75bpm. What is her stroke volume? A. 73.33 ml B. 206.25 ml C. 13.64 ml D. 69.5 ml E. 412 ml

- answer is A

What lead primarily measures forces moving from the head to the feet? A. Lead I B. aVF C. V1 D. V6 E. aVR

- answer is B Lead II, III and aVF measure from head to feet. aVF is the only choice provided

What effect does prolonged training have on resting heart rate? A. Arrhythmic B. Decreased C. No change D. Increased

- answer is B. Decreased

Cardiac muscle acts as a syncytium due to the presence of ___ and ___ found in intercalated discs A. sarcomeres, dyad B. t-tubules, sarcoplasmic reticulum C. desmosomes, gap junctions D. mitochondria, dyad

- answer is C

Because of the placement of the positive electrodes, and the predominant direction of the depolarization wave of the heart, the QRS complex is ________ in lead I and II. A. neutral B. isoelectric C. positive D. negative

- answer is C. Positive

What effect does prolonged training have on VO2 max? A. Decrease B. Arrhythmic C. No change D. Increase

- answer is D

The corresponding pressure during the beginning of isovolumetric contraction is quite ___________________ because the ventricular muscle is relaxed. A. high B. low

low

The ventricular _______________________ loop describes one complete cycle of ventricular contraction, ejection, relaxation, and refilling.

pressure-volume

Ohm's Law: △P = F x R Hydrostatic pressure (P): pressure exerted by the fluid (blood) △P = ____________ (mmHg)

P1-P2 (△P = MAP - right atrial pressure)

Which of the following senses the intracellular concentration of calcium in smooth muscle? A.) Calmodulin B.) Myosin C.) Troponin D.) MLCK E.) Actin

Calmodolin

According to ohms law, deltaP=F*R. Flow to the pulmonary vasculature is equal to that of the systemic vascular. How does the pulmonary vasculature accommodate the same amount of blood as the systemic vasculature? A. Increased Resistance B. Increased Velocity C. Decreased Resistance D. Decreased Viscosity E. Increased Pressure

answer is C R = P/flow pulmonary circulation - has low pressure, high compliance. It also normally has a low resistance - pulmonary arterial pressure - left atrial pressure = 15 - 5 = 10 mmHg (pressure for pulmonary circulation) - systemic arterial pressure - right atrial pressure = 100 - 2 = 98 mmHg (pressure for systemic circulation) Since blood flow is the same, therefore resistance of pulmonary circulation is 1/10th the resistance of systemic circulation.

Arrange the following in the proper order in which they occur at the pre-synaptic side of a neuromuscular junction. 1. Calcium ions are released 2. Action potential arrives at the presynaptic terminal 3. Neurotransmitter is released A) 1, 2, 3 B) 2, 1, 3 C) 2, 3, 1 D) 3, 2, 1 E) 3, 1, 2

B) 2, 1, 3

Energy is released when... A) ADP is broken down into ATP and phosphate. B) ATP is broken down into ADP and phosphate. C) ADP and phosphate combine to form ATP. D) ATP and phosphate combine to form ADP. E) AMP and two phosphates combine to form ATP.

B) ATP is broken down into ADP and phosphate

Which of the following statements is true? A) The longer the vessel, the lower the resistance and the greater the flow. B) As blood volume decreases, blood pressure and blood flow also decrease. C) Increased viscosity increases blood flow. D) All of the above are true.

B) As blood volume decreases, blood pressure and blood flow also decrease.

Which of the following best describes veins? A) thick walled, small lumens, low pressure, lack valves B) thin walled, large lumens, low pressure, have valves C) thin walled, small lumens, high pressure, have valves D) thick walled, large lumens, high pressure, lack valves

B) Thin walled, large lumens, low pressure, have valves (large lumen = large diameter = large radius)

An action potential enters a neuron... A) by diffusion. B) by the T-tubules. C) by the sarcolemma. D) by the sarcoplasmic reticulum. E) by the actin myofilament.

B) by the T-tubules

During contraction of a muscle, calcium ions bind to.. . A) the actin myofilament. B) the troponin molecule. C) the tropomyosin molecule. D) the sarcoplasmic reticulum. E) the sarcolemma.

B) the troponin molecule

What structures do the calcium ions bind to when muscle contraction is initiated? A) the actin myofilament. B) the troponin molecule. C) the tropomyosin molecule. D) the sarcoplasmic reticulum. E) the sarcolemma.

B) the troponin molecule

Which of the following statements about the "heads" of the myosin molecules is TRUE? A) they form permanent links with actin filaments B) they can attach to different sites on the actin filament C) they straighten the myosin filaments D) they allow the myosin filaments to wrap around each other E) none of the above

B) they can attach to different sites on the actin filament

Each actin filament is composed of A) a single strand of actin molecules. B) two strands of actin molecules wrapped together. C) three strands of actin molecules wrapped together. D) four strands of actin molecules wrapped together. E) hundreds of strands of actin molecules wrapped together.

B) two strands of actin molecules wrapped together

Arrange the following in the proper order in which they occur at the post-synaptic side of a neuromuscular junction. 1. Action potential is propagated over the muscle cell membrane 2. Depolarization of the post-synaptic membrane 3. Sodium ions move into muscle cell A) 1, 2, 3 B) 2, 1, 3 C) 2, 3, 1 D) 3, 2, 1 E) 3, 1, 2

D) 3, 2, 1

The isovolumetric ventricular relaxation and the ventricular filling phases are parts of the ventricular _______________ phase of the cardiac cycle. A. Systole B. Diastole

Diastole

Muscle contraction is caused by A) actin filaments sliding past each other. B) myelin filaments sliding past each other. C) myosin filaments sliding past each other. D) actin filaments sliding past myelin filaments. E) actin filaments sliding past myosin filaments.

E) actin filaments sliding past myosin filaments.

As actin and myosin filaments slide past each other during muscle contraction, A) actin filaments shorten, while myosin filaments do not. B) myosin filaments shorten, while actin filaments do not. C) either actin or myosin filaments shorten, but not both at the same time. D) both actin and myosin filaments shorten. E) neither actin nor myosin filaments shorten.

E) neither actin nor myosin filaments shorten

Cross bridges form between... A) troponin and tropomyosin. B) calcium and sodium. C) the sarcolemma and the sarcoplasmic reticulum. D) the T-tubules and the sarcolemma. E) the actin filaments and the myosin heads.

E) the actin filaments and the myosin heads

Which of the following will tend to increase venous return? A) Hemorrhage B) Breathing Out C) Increased ventricular contractility D) Standing E) Increased Right Atrial Pressure

Increased ventricular contractility ( increase CO = increase VR)

High altitude leads to decreases Pos, how does this affect pulmonary vascular resistance? A. increases B. decreases C. no effect

Increases

As venous return increases, end-diastolic volume increases, and because of the length-tension relationship in the ventricles, _______________________ increases accordingly.

stroke volume

The isovolumetric ventricular contraction and the ventricular ejection phases are parts of the ventricular ___________ phase of the cardiac cycle. A. systole B. diastole

systole

At the beginning of ____________________ , the left ventricular pressure becomes higher than aortic pressure which causes the aortic valve to open. A. isovolumetric contraction B. ventricular ejection C. isovolumetric relaxation D. Isovolumetric relaxation

ventricular ejection

During ventricular ejection, ventricular _______________________ decreases dramatically as blood is ejected into the aorta. A. pressure B. volume

volume

If a persons SV was 70 ml, EDV is increases from 135 to 165ml, without any changes in arterial pressure. What is the SV in the next few cardiac cycles

- 100 ml

In which phase of the ventricular muscle action potential is the potassium permeability highest? A. 1 B. 2 C. 3 D. 0 E. 4

- 3

What percentage of blood volume is pumped by the R ventricle of the fetus? A. 34 B. 100 C. 59 D. 90 E. 66

- 66% systemic cardiac output is combination of left and right ventricles, with the right ventricle actually contributing more than the left to the total (approximately 65% vs. 35%), 50% of the fetal blood volume is in the placenta and 50 % in the body. 2. Blood flow is the movement of blood through a vessel, tissue, or organ.i n the arterial system, as resistance increases, blood pressure increases and flow decreases. In the venous system, constriction increases blood pressure as it does in arteries; the increasing pressure helps to return blood to the heart.pulmonary circulation is a system of low resistance, about one-sixth that of the systemic circulation

Be able to draw the sarcomere. Label each zone and each fiber and protein associated with each zone. Describe how the zones change when contracted.

- Basic structural & functional unit of skeletal muscle because it is the smallest portion of skeletal muscle capable of contracting. - Z disc to Z disc - I band - made up of thin filaments, Titin and Z discs - A band - thick and thin filaments - H zone - thick filament - M line - When a muscle contracts, the sarcomere shortens

Why is the first breath so important? What happens in the lungs? What is surfactant?

- The first breath of life is really hard because the baby has to generate a very large negative pressure to try to expand its lungs - At birth the first breath brings in oxygen and relieves the hypoxic vasoconstriction allowing pulmonary blood flow to greatly increase - - first breath: lung is stiff & needs to increase pressures in the lung, decrease in pulmonary vascular resistance - second breath: easier, lungs have matured, pseumocytes have created pulmonary cirfactin -- decrease surface area in alveoli - 40 minutes: inflammation of the lung is much easier, effective

Just prior to atrial systole, what electric event occurs? A. P-wave B. T-Wave C. QRS-Wave

- answer is A

What triggers the opening of the mitral valve? A. Pressure in the ventricle drops below the atrial pressure B. Pressure in the ventricle rises above the atrial pressure C. Pressure in the ventricle rises above the aortic pressure

- answer is A

T wave of the ECG best represents which heart event? A. Ventricular repolarization B. Ventricular contraction C. Ventricular relaxation D. Ventricular depolarization

- answer is A - B is wrong. QRS complex - D is wrong. Represents QRS

Becker's Muscular dystrophy is due to ______________. A. An in frame mutation in the dystrophin gene B. methylation of the dystrophin gene C. post translation modifications of dystrophin protein D. An out of frame mutation in the dystrophin gene

- answer is A - D is wrong this is for Duchenne - the more lethal form

Smooth Muscle fibers are similar to cardiac and skeletal muscle fibers in that they have all of the following EXCEPT A. Troponins B. Tropmyosin C. Myosin Filaments D. Actin Filaments

- answer is A - have thick and thin & tropomyosin - lack troponin 1. Ca binds to comodulin 2. activates myosin light chains kinase & phosphorylates regulatory light chains 3. changes orientation of mysoin heads & to heads reach around to tropomyosin & binds to active sites 4. undergoes crossbridge cycle - think filaments - troponins, tropomysoin, and actin filaments - thick filametns - myosin filaments and titin

In a normal healthy person an increase in cardiac output will decrease: A. Venous return B. Pulmonary vascular resistance C. Pulmonary arterial pressure D. Systemic arterial pressure E. The size of zone 3 of the lung

- answer is B Increase in cardiac output would decrease pulmonary vascular resistance because it would increase the blood volume to the lungs, this leads to recruitment and distension and reduction in PVR

The latch property of smooth muscle refers to the ability of the smooth muscle to maintain: A. entirely relaxed state independent of free Ca++ levels B. force production while decreasing the rate of energy utilization C. the rate of ATP production after stimulation D. force in the absence of myosin light-chain phosphorylation E. a decreased membrane potential

- answer is B Latch" is the ability of smooth muscle to maintain force production while decreasing the rate of energy utilization. This is how blood vessels and sphincters are able to maintain long contractions without fatiguing.

Three tubes are arranged in parallel, as shown in the following illustration: The resistance to flow in tube 1 is 100 mm Hg/(L/min), the resistance to flow in tube 2 is 50 mm Hg/(L/min), and the resistance to flow in tube 3 is 25 mm Hg/(L/min). The total resistance of these three tubes arranged in parallel is closest to which of the following choices? A. 0.06 mm Hg/(L/min) B. 14.3 mm Hg/(L/min) C. 26.7 mm Hg/(L/min) D. 58.3 mm Hg/(L/min) E. 175 mm Hg/(L/min)

- answer is B The Law of LaPlace explains tension in vessel walls, aneurysms burst when they lose that tension ratio.

Of the following sites within the fetal circulation, which has the highest oxygen saturation? A. umbilical artery B. umbilical vein C. descending aorta D. carotid artery E. superior vena cava

- answer is B The umbilical vein comes from the mother, and she supplies the oxygen, thus the blood here has the highest oxygen content. It will steadily lose oxygen as it travels through the fetus.

The fick principle can be used to determine A. metabolic rate B. All of these C. Oxygen consumption D. cardiac output

- answer is B all of these

Why is the "funny current" important for cardiac conductance? A. Prolongs the Absolute Refractory Period B. Creates the Pacemaker Potential C. Rapid Re-polarization D. Rapid Depolarization E. Stabilization of the Transmembrane Potential

- answer is B. Funny current is important in slow response action potentials.

What is the consequence of increased activation of beta 1-adrenergic receptors in ventricular cardiomyocytes? A. Decreased Ca++ permeability and Increased contractility B. Increased Ca++ permeability and contractility C. Decreased Ca++ permeability and contractility D. Increased Ca++ permeability and decreased contractility

- answer is B. Increased Ca++ permeability and contractility

Increased shear stress across the endothelial cell wall leads to increased production of which molecule? A. CO2 B. NO C. Glucose D. H+ E. O2

- answer is B. NO

During exercise, this mechanism is responsible for the initial increase in heart rate. A. Sympathetic Withdrawal B. Parasympathetic Withdrawal C. Sympathetic Activation D. Parasympathetic Activation

- answer is B. Parasympathetic Withdrawal

At the end of atrial systole _______. A. Pressure in the ventricle is lower than the pressure in the atrium B. Pressure in the atrium is greater than pressure in the ventricle C. Pressure in the ventricle rises above atrial pressure

- answer is C

What phase occurs after pressure in the ventricle drops below the atrial pressure? A. Slow ventricular filling phase B. Rapid Ejection phase C. Rapid ventricular filling phase

- answer is C

Mean arterial pressure readings over 3 months are between 90 and 100 mmHg for patient X and between 150 and 160 for patient Y. A rapid decrease in carotid sinus pressure would: A. Decrease both sympathetic and parasympathetic nerve activity only in patient X B. Increase sympathetic and decrease parasympathetic nerve activity only in patient X C. Decrease parasympathetic nerve activity about equally in both patients D. Stimulate the chemoreceptor reflex in both patients E. Increase plasma renin activity only in patient Y

- answer is C The baroreceptor reflex works on the same curve, but the resetting effect allows the reflex to work within the same sigmoidal shape, but at different levels of pressure for different individuals.

Access to the actin active site is regulated by __________ in smooth muscle cells and _____________ in cardiac and skeletal muscle. A. Calcium binding troponin, Myosin phosphorylation B. Calcium binding tropomyosin, Calcium binding troponin C. Myosin phosphorylation, calcium binding troponin D. Myosin phosphorylation, calcium binding tropomyosin

- answer is C smooth muscle lacks troponin so A is wrong

A Sarcomere is measured from A. I Band to I Band B. A Band to A band C. Z Disk to Z disk D. M line to M line E. none of the above

- answer is C A Sarcomere is measured from: Z-disk to Z-disk.

Which of the following will increase stroke volume? A. Increased arterial pressure B. Reduced activity of cardiac sympathetic nerves C. Increased ventricular filling pressure D. Low extracellular Ca2+ E. Reduced end-diastolic volume

- answer is C. - Increased arterial pressure would increase afterload which would decrease stroke volume -Reduced activity of cardiac sympathetic nerves would decrease contractility and thus stroke volume *Increased ventricular filling pressure would increase stretch on the myocardium and increase stroke volume (Increase preload/Frank-starling law) - Low extracellular Ca2+ would decrease contractile strength and thus stroke volume - Reduced end-diastolic volume would reduce pre-load and thus stroke volume

A patient has entered the ER with a gunshot would to the leg. It has taken 2 hours for the ER to the patient from the time they were intitially shot. The BP is 90/60 and the hear is Bradycardic. You note the patient's skin is pale and cool, their breathing is slow and shallow. What stage of shock is this patient in? A. Compensated B. No shock C. Decompensated D. Reversible

- answer is C. Decompensated

Babies born from mothers who experienced nutrient deprivation during gestation at risk for what disease later in life? A. Pompe Syndrome B. Addison's disease C. Hypertension D. Muscular Dystrophy E. Epilepsy

- answer is C. Hypertension

Despite fluctuations to systemic blood pressure, blood flow is maintained relatively constant to the brain. What is the mechanism for this phenomenon? A. Increased Baroreceptor Activation B. Decreased Renin Angiotensin System Activity C. Metabolic Autoregulation D. Decreased Baroreceptor activation E. Increased NO production

- answer is C. Metabolic Autoregulation

Isovolumetric contraction of the left ventricle A. begins when the aortic valve opens and ends when the aortic valve closes B. is not influenced by changes in aortic pressure C. begins when mitral valve closes and ends when the aortic valve opens E. requires less time (is shorter) than isovolumetric contraction of the right ventricle

- answer is C. begins when mitral valve closes and ends when the aortic valve opens

PC = 35 pc = 14 pi = 3 Pi = 0 If the NDP above is on the arterial end of the capillary, and the only change in starling forces on the venous end are a decrease in capillary hydrostatic pressure to 15, what will the net direction of fluid? A. Fluid will flow into the capillary B. No net fluid movement C. Fluid will flow out of the capillary

- answer is C. fluid will flow out of the capillary

A patient with left heart failure has pulmonary edema. This is primarily caused by A. systemic hypertension B. increased SV C. increased end diastolic volume D. increased end systolic volume E. increased total peripheral resistance

- answer is C. increased end diastolic volume

A patient has entered the ER with a gunshot would to the leg. It has taken 1 hour for the paramedics to get him to the ER from the time that he was initially shot. According to the paramedics, the patient has been steadily losing blood; however, blood pressure is still in the normal range (110/75) This patients heart rate will be A. fibrillating B. blocked C. tachycardic D. normal sinus rhythym E. bradycardic

- answer is C. tachycardic

A hiker is climbing a mountain, and his chemoreceptors have started firing. What has caused this increase in chemoreceptor activity? A. Decreased PaCO2 B. Increased respiration due to exercise/hiking C. Increased PaO2 D. Decreased PaO2 E. Decreased H+ ion concentration in the Blood

- answer is D

Calcium is released and sequestered differently in skeletal and cardiac muscles. What is the physiological relevance of skeletal muscle calcium handling? A. Relies on Calcium Induced Calcium Released for regulation of contractile strength B. Calsequestrin prevents tetany C. Phospholambin increases Calcium reuptake D. Large Calcium stores in the SR allows for full contraction once activated

- answer is D

In an individual with a heart rate of 50 beats/ min, stroke volume of 80ml, an arterial blood pressure of 140/80 mm Hg, and a right atrial pressure of 0mm Hg, what is the total peripheral resistance? A. 15 mmHg*minL-1 B. 10 mmHg*minL-1 C. 5 mmHg*minL-1 D. 25 mmHg*minL-1 E. Not enough information to determine

- answer is D

Which of the following relations or laws fundamentally explains why aneurysms burst? A. Ohm's Law B. Starling's Law of the Heart C. Pouiseille's Relation D. LaPlace's Law E. Murphy's Law

- answer is D

Which vessel is the most compliant? A. Aorta B. Arterioles C. Capillaries D. Venules E. Vena Cava

- answer is D

You give a drug intravenously that you think is a selective alpha 1 receptor agonist to a patient. you would expect to observe the following direct effects: A. Increased Stroke Volume B. Increased cardiac output C. Increased SV and CO D. Increased total peripheral resistance E. Decreased heart rate

- answer is D

During inspiration extra-alveolar resistance ________ and alveolar resistance ___________

- decreases, increases Extra-alveolar vessels in the pleural space and not surrounded by alveoli Alveolar vessels small arterioles in the capillaries that are embedded within the alveoli

The membrane electrical potential at which the inward flow of an ion is equal to its outward flow best describes a given ion's ______________ A. equilibrium potential B. Nernst number C. diffusional gradient D. concentration gradient E. voltage

- equilibrium potential

During contraction muscle fibers fatigue at different rates. What is ultimately the cause of muscle fatigue? A. Increased Glycolysis B. Increased Lactic Acid C. Loss of Oxygen D. Loss of ATP

- the answer is B

What causes pulmonary edema?

- when pulmonary arterial and/or venous pressure increases, more fluid enters the interstital space. If the lymphatic system cannot absorb the fluid, the result is Pulmonary edema (excessive accumulation of fluid in tissue space, in this example more fluid in interstitial space then can flow by lymphatic flow) - elevations in pulm arterial or venous pressure - increased capillary permeability - impaired lymphatic flow or hemodilution can push out of the capillaries & into the interstitial space & alveoli - caused by left heart failure - primarily caused by increased end diastolic volume - left heart failure --> back up fluid in pulm veins & caps --> increased pulm cap Pc --> increased filtration --> pulmonary edema

In an individual with a heart rate of 50 beats/min, stroke volume 80 ml, an arterial blood pressure of 140/80 mm Hg and a right atrial pressure of 0 mm Hg, what is the total peripheral resistance? A) 15 mmHg*min*L-1 B) 10 mmHg*min*L-1 C) 25 mmHg*min*L-1 D) 5 mmHg*min*L-1 E) Not enough information to determine

25 mmHg*min*L-1 [ 🔺P = F x R ] [* 🔺P = P1 - P2 = MAP - right atrial pressure ] [* MAP = 1/3 SP + 2/3 DP = 1/3 (140) x 2/3 (80) = 100 ] [* F = CO = SV x HR = 80 x 50 = 4 L ] [ Rv = (MAP - right atrial pressure) ➗cardiac output ] [ Rv = 100 - 0 ➗4 = 25 ]

The neurotransmitter in the neuromuscular junction... A) causes ligand gated calcium channels in the muscle fiber to increase their permeability to calcium, which depolarizes the postsynaptic membrane. B) causes ligand gated sodium channels in the muscle fiber to increase their permeability to sodium, which depolarizes the postsynaptic membrane. C) increases the calcium ion permeability of the presynaptic membrane. D) increases the sodium ion permeability of the presynaptic membrane. E) depolarizes the presynaptic membrane by increasing the permeability of sodium ions.

A) causes ligand gated calcium channels in the muscle fiber to increase their permeability to calcium, which depolarizes the postsynaptic membrane.

In the neuromuscular junction, acetylcholine... A) diffuses across the synaptic cleft and binds to acetylcholine receptors on the postsynaptic muscle fiber. B) is actively transported across the synaptic cleft and binds to acetylcholine receptors on the postsynaptic muscle fiber. C) diffuses across the synaptic cleft and causes calcium ions to diffuse into the postsynaptic muscle fiber. D) diffuses across the synaptic cleft and causes sodium ions to diffuse out of the postsynaptic muscle fiber. E) is actively transported across the synaptic cleft and causes calcium ions to diffuse into the postsynaptic muscle fiber.

A) diffuses across the synaptic cleft and binds to acetylcholine receptors on the postsynaptic muscle fiber.

During muscle contraction A) the A band remains the same and the I band narrows. B) the I band remains the same and the A band narrows. C) both the A band and I band remain the same. D) both the A band and I band narrow. E) both the A band and I band become wider.

A) the A band remains the same and the I band narrows.

Which of the following statements about H zones and I bands is TRUE? A) the H zones contain only myosin, while the I bands contain only actin B) the H zones contain only actin, while the I bands contain only myosin C) the H zones contain only myosin, while the I bands contain both actin and myosin D) the H zones contain both actin and myosin, while the I bands contain only actin E) the H zones and the I bands each contain both actin and myosin

A) the H zones contain only myosin, while the I bands contain only actin

When blood flows into the R ventricle it must pass through which valve? A. Tricupsid B. Bicupsid C. Pulmonic D. Aortic E. Mitral

Tricuspid - B and E is wrong, from the left atrium to left ventricle - C is wrong, from the right ventricle to the pulmonary artery to the lungs - D is wrong, from the left ventricle to the aorta to the body (systemic circulation)

___________________ is ventricular wall stress during systole or ejection.

afterload

Venoconstriction increases which of the following? A) blood pressure within the vein B) blood flow within the vein C) return of blood to the heart D) all of the above

all of the above

Which of the following vessels has the highest resistance? A) venuoles B) arterioles C) capillaries D) veins E) Aorta

arterioles

Just prior to atrial systole, what electric event occurs? A) P-wave B) T-Wave C) QRS-Wave

P-wave

What triggers the opening of the mitral valve? A. Pressure in the ventricle drops below the atrial pressure B. Pressure in the ventricle rises above the atrial pressure C. Pressure in the ventricle rises above the aortic pressure

Pressure in the ventricle drops below the atrial pressure

At the end of atrial systole _______. A) Pressure in the ventricle is lower than the pressure in the atrium B) Pressure in the atrium is greater than the pressure in the ventricle C) Pressure in the ventricle rises above atrial pressure

Pressure in the ventricle rises above atrial pressure

The purpose of the ductus arteriosus is to shunt blood from the __________ to the _____________

R ventricle, aorta - the blood in the pulmonary artery, only some of that goes into the lungs and most of it goes through a shunt called the arterial duct (or ductus arteriosis) which takes blood from the pulmonary artery into the arch of the aorta and dumps the blood into the systemic circulation

What phase occurs after pressure in the ventricle drops below the atrial pressure? A. Slow ventricular filling phase B. Rapid Ejection phase C. Rapid ventricular filling phase

Rapid ventricular filling phase

Which of the following channels allows the movement of calcium from outside the cell directly into the sarcoplasmic reticulum? A.) SERCA B.) Na-Ca exchangers C.) Na-K ATPase D.) Store-operated channels

Store-operated channels

Understand the Zones of the lung - Where are they usually located? - What causes the zonation? Functional not anatomical

The zones are defined functionally and not anatomically - Ppa: pulmonary arterial pressure - Ppv: pulmonary venous pressure - Pa: alveolar pressure 1) Zone 1: - Pa> Ppa > Ppv - Alveolar pressure is greater than either pressure, so therefore it is essentially squishing the pulmonary vessels closed and there is blood flow through neither of the vessels - not present during normal conditions - can occur during hemorrhage 2) Zone 2: - Ppa > Pa > Ppv - Happens in the middle of the lungs - The pulmonary arterial pressure is high enough to force some blood flow through the lungs, but it is limited by the fact that alveolar pressure exceeds pulmonary venous pressure and tends to close the vessels towards the venous side of the circulation 3) Zone 3: - Ppa > Ppv> Pa - The most blood flow takes place in this zone, towards the base of the lungs - Blood flow through these vessels is similar to blood flow through any vessel in the systemic circuit - Alveolar vessel resistance is lowest in this zone

Ohm's Law: △P = F x R ⇢F = △P/R Flow (F): rate of fluid movement = __________________

cardiac output (CO = SV x HR)

Increasing right atrial pressure will ________________ cardiac output. A) increase B) decrease

decrease (*** Ohm's Law: 🔺P = Q x R **** * Right atrial pressure = downstream pressure * Cardiac output = flow)

Breathing out will ______________ venous return. A) increase B) decrease

decrease (pleural pressure)

A patient was born with a congenital heart defect called "Coarctation of the Aorta," in which the aorta narrows at a specific point in the arch, followed by an immediate return to normal diameter size. (Think of it as if a rubber band is restricting the middle of the aortic arch). This restriction point in the aortic arch would ____________ blood flow? A) increase B) decrease

decrease (radius ↓: ↑ resistance, ↓ flow) (The left ventricle has to work harder to force blood through the narrowed part of the aorta )

Negative inotropic agents produce a(n) _______________________ ejection fraction. A. increased B. decreased

decreased

During isovolumetric relaxation, ventricular pressure _________________ but volume remains constant A. increases B. decreases

decreases

When blood pressure decreases, afterload _____________________. A. increases B. decreases

decreases

When peripheral vascular resistance decreases, afterload ___________________________. A. increases B. decreases

decreases

If Pressure is constant: as radius ↓, resistance ↑, and flow _____________.

decreases (radius ↓, resistance ↑, flow ↓)

Isovolumetric contraction begins at the end of _________________ a. systole b. diastole

diastole

The Frank-Starling relationship states that the volume of blood ejected by the ventricle depends on the volume present in the ventricle at the end of _________________.

diastole

In a blood pressure measurement of 110/70, the number 70 is the __________________. A) systolic pressure B) diastolic pressure C) pulse pressure D) mean arterial pressure

diastolic pressure

Closer to the heart, arteries would be expected to have a higher percentage of ____________________. A. endothelium B. smooth muscle fibers C. elastic fibers D. collagenous fibers

elastic fibers

In the steady state, cardiac output ______________________ venous return. A. equals B. is greater than C. is less than

equals

Ohm's Law: △P = F x R Resistance (R): force exerted by the tube (vessel) that regulates pressure and flow * Resistance is inversely proportional with _______________

flow

During ventricular ejection, left ventricular pressure remains __________ because the ventricle is contracting. A. high B. low

high

Slight vasodilation in an arteriole prompts a __________________. A) slight increase in resistance B) huge increase in resistance C) slight decrease in resistance D) huge decrease in resistance

huge decrease in resistance

Positive inotropic agents produce ____________________ in stroke volume and cardiac output for a given end-diastolic volume. A. increased B. decreased

increased

Positive inotropic agents produce a(n) _______________________ ejection fraction. A. increased B. decreased

increased

If Pressure is constant: as length ↓, resistance ↓, and flow _____________.

increases (length ↓, resistance ↓, and flow ↑)

If Pressure is constant: as viscocity ↓, resistance ↓, and flow _____________.

increases (viscosity ↓, resistance ↓, and flow ↑)

A healthy elastic artery __________________. A) is compliant B) reduces blood flow C) is a resistance artery D) has a thin wall and irregular lumen

is compliant

In the isovolumetric ventricular relaxation and the isovolumetric ventricular contraction phases there _______ blood flowing through the valves. A. is B. is no

is no

When the ventricle contracts during ____________________ the ventricular pressure increases dramatically since all valves are closed. A. isovolumetric contraction B. ventricular ejection C. isovolumetric relaxation D. Isovolumetric relaxation

isovolumetric contraction

At the beginning of ____________________, systole ends and the ventricle relaxes. A. isovolumetric contraction B. isovolumetric relaxation

isovolumetric relaxation

For simplicity's sake, it is assumed that aortic pressure during ejection equals _______________________ pressure during ejection.

left ventricular

The mitral valve opens when the ventricular pressure is ___________ the atrial pressure. A. less than B. greater than

less than

The ionic gradient across a heart cell membrane at rest favors K+ moving _____________ and Na+ moving ______________ the cell

outside, inside

Poiseuilles Law: △P = F x k♏︎(l/r^4) Is the interrelationship among pressure, flow and the 3 components of __________________

resistance - radius, length, viscosity

After the rapid ventricular filling phase, ventricular pressure will _______. A. rapidly rise toward aortic pressure B. slowly rise toward atrial pressure C. stay the same

slowly rise toward atrial pressure

If we assume that aortic pressure during ejection is equal to left ventricular pressure during ejection, then we can say that left ventricular pressure during ejection is equal to what we commonly know as ________________________________. A. systolic blood pressure. B. diastolic blood pressure

systolic blood pressure.

In skeletal muscle, when there is maximal load, the velocity of contraction equals 0. This is considered what type of contraction? A. Shortening B. Lengthening C. Isotonic D. Isometric

the answer is D isometric shortening: isotonic - shortening against fixed load, speed, dependent on MATPase actigvity & load (sarcomeres brought to center & msk shortened) 2. isometric: no change in length, m-heads are cycling & generating tension 3. lengthening: myosin heads try to pull thin filaments on top of thick filaments, can cause msk damage, posing force is greater

The ventricular volume present at the end of diastole depends on compliance and the ___________________.

venous return

At the beginning of ______________________ , ventricular pressure has fallen to a level that now is less than left atrial pressure, causing the mitral valve to open. A. ventricular filling B. ventricular ejection

ventricular filling

Preload is defined as left ventricular _________________ at the end of diastole. A. pressure B. volume C. wall stress

wall stress

Skeletal Muscle

- cross striations and voluntary

What is one important benefit to the parallel arrangement of vasculature A. Additive Resistance B. Individual control of blood flow to specific organs and tissues C. Decreased total peripheral resistance D. A and B E. B and C F. A, B, and C

- answer is E - A is wrong, this is series arrangement. Total peripheral resistance will increase.

Calcium is released and sequestered differently in skeletal and cardiac muscles. What is the physiological relevance of cardiac muscle calcium handling? A. Relies on Calcium Induced Calcium Released for regulation of contractile strength B. Calsequestrin prevents tetany C. Phospholambin increases Calcium reuptake D. Large Calcium stores in the SR allows for full contraction once activated

- answer is A CICR is necessary for cardiac muscle contraction and is found in extracellular fluid CICR is not necessary for skeletal --> no external Ca++ In cardiac muscle there is an absolute requirement of extracellular calcium - B is wrong. The force of contraction can be increased by activating more motor neurons (recruiting more msk fibers) or by increasing the frequency of AP in the msk fibers, which produces tetany. The increased force during tetanic contractions is due to prolonged elevation of intracellular Ca++. - D is wrong. That answer is for skeletal muscle

Which of the following, in the absence of changes in other variables, will tend to decrease venous return? A. increased right atrial pressure B. decreased venous compliance C. increased blood volume D. decreased venous resistance E. increased mean circulatory filling pressure

- answer is A Increased Right atrial pressure decreases the driving pressure (if nothing else changes) and makes it harder for blood to return to the heart. Thus decreasing venous return

If you give a patient spinal anesthesia you would expect: A. A decrease in HR and a decrease in BP B. An increase in HR and increase in BP C. A decrease in HR and no change in BP D. A decrease in HR and an increase in BP E. An increase in HR and a decrease in BP

- answer is A - A decrease in HR and a decrease in BP

When compared to events on the left side of the heart, the right side A. Develops less ventricular pressure during systole B. Has a lower stroke volume C. Has a higher preload D. Generates heart sounds S3 and S4 E. Depends upon atrial contractions for adequate diastolic filling

- answer is A because systole and diastole pressure, so the mean blood pressure, decrease as we go form aorta -> arteries -> arterioles -> capillaries -> venules -> veins -> vena cava - B is wrong. They have the same stroke volume - C is wrong because they have same preload. Increased preload will increase stroke volume

What part of the circulatory system has the largest cross-sectional area? A. Capillaries B. Aorta C. Greater Arteries D. Arterioles E. Greater Veins

- answer is A, they are responsible for a decrease in blood velocity and resistance - D is wrong. Arterioles represent the site of greatest resistance. They are considered as the primary resistance vessels as they distribute blood flow into capillary beds. Arterioles provide approximately 80% of the total resistance to blood flow through the body. - E is wrong. They store the most blood

Pc = 35 pc = 14 pi = 3 Pi = 0 What is NDP? A. 24 B. 52 C. 49 D. 11 E. 14

- answer is A. 24

What would be the heart rate range of a patient whom has experienced a block of conduction from the SA node? A. 40-60 bpm B. 35>bpm C. 60-100 bpm D. 100-120 bpm

- answer is A. 40-60bpm the heart will default to contracting at ventricular "pacemaker" pace without signaling from the SA node. No atrial contraction.

What is the correct order of the chemoreflex pathway? A. Chemoreceptors -> Vagus Nerve -> NTS -> RVLM -> Increased SNS activity B. Chemoreceptors -> Vagus Nerve -> NTS -> CVLM -> Decreased SNS activity C. Chemoreceptors -> Vagus Nerve -> NTS -> CVLM -> Increased SNS activity D. Chemoreceptors -> Vagus Nerve -> NTS -> RVLM -> Increased PSNS activity E.Chemoreceptors -> Vagus Nerve -> NTS -> NA -> Increased SNS activity

- answer is A. Chemoreceptors -> Vagus Nerve -> NTS -> RVLM -> Increased SNS activity

The RYR is a __________ release channel found in the _____ of the _________ A. Ca+, sarcoplasmic reticulum, triad B. Ca+, t-tubule, myofibril C. Na+, t-tubule, myofibril D. Na+, Sarcoplasmic reticulum, triad

- answer is A: Ca+, sarcoplasmic reticulum, triad

A 30-year-old man had his EKG measured at his physician's office, but his records were lost. The EKG technician remembered that the QRS deflection was large and positive in lead aVF and 0 in lead I. What is the mean electrical axis? A. not enough information B. 90 C. 45 D. 30

- answer is B

After the rapid ventricular filling phase, ventricular pressure will _______. A. rapidly rise toward aortic pressure B. slowly rise toward atrial pressure C. stay the same

- answer is B

If a patient is presenting with a right bundle branch block, what does this mean for the mechanical function of the heart? A. Right ventricle will contract before the left ventricle B. Left ventricle will contract before the right ventricle C. Ventricles will contract in syncytium D. Atria will not contract

- answer is B

The QRS complex is predominantly negative in this lead of the normal 12 lead ECG. A. aVL B. aVR C. III D. I E. II

- answer is B

What is the percent oxygenation saturation level in the fetus thoracic aorta? A. 70 B. 65 C. 60 D. 55 E. 50

- answer is B - A is wrong: Represents IVC - D is wrong: Right Ventricle

If hemorrhage causes precapillary resistance arterioles to constrict and capillary hydrostatic pressure decreases, which of the following is most likely to occur? A. Net filtration of fluid from the capillaries will increase B. Net absorption of fluid in the capillaries will increase C. Net absoprtion of fluid in the capillaries will not change D. Capillary diameter will decrease E. Capillary diameter will increase

- answer is B A decrease in arterial capillary hydrostatic pressure would reduce the amount of fluid pushed out of the capillary and thus reduce net filtration and increase net absorption

Which of the following would be most likely to occur if the AV node was damaged by ischemia? A. The ability to intrinsically increase heart rate during stress (such as exercise) would be impaired. B. Conduction of the depolarizing electrical signal from the atria to the ventricles would be delayed or blocked. C. Spreading of the depolarizing electrical signal from the septum to the ventricular walls would be impaired. D. Cardiac contractility would be impaired. E. Cardiac contractility would be increased

- answer is B If the AV node was damaged by ischemia it would result in a block of the conduction from the AV node to the bundle of His.

Jim is lifting weights. Holding a 20lb weight down by his side, he bends his arm at the elbow, contracting his bicep, and holds the position until he reaches muscle fatigue. According to the Henneman Size Principle in what order will his muscle fibers be recruited? A. Fast Twitch Type 2a > Slow Twitch > Fast Twitch Type 2b B. Fast Twitch Type 2b > Fast Twitch Type 2b > Slow Twitch C. Fast Twitch Type 2b > Slow Twitch Fast Twitch Type 2a D. Slow Twitch > Fast Twitch Type 2a > Fast Twitch Type 2b E. Fast Twitch Type 2a > Fast Twitch Type 2b > Slow Twitch

- answer is D - starting with the smallest motor unit, progressively larger units are recruited with increasing strength of muscle contraction - the result if an orderly addition of motor units resulting in smooth increase in muscle strength Type I: Slow twitch - fatigue resistent - color: red - metabolism: oxidative - mitochondria: high - glycogen low Type IIa: Fast - fatigue resistent - red - metabolism: oxidative - mitochondria: higher - glycogen abundant Type IIb: Fast - fatiguable - color: white: low myoglobin - metabolism: glycolytic - fewer mitochondria - high glycogen

How would the presence of an insufficient aortic valve (prolapse in which the valve does not close properly and can allow backflow into the ventricle) affect the pump function of the heart? A. Immediately decrease contractility and cardiac output B. Immediate ventricular fibrillation C. Compensatory decrease in contractility to increase cardiac output D. Compensatory increase in contractility to maintain cardiac output

- answer is D A prolapsed aortic valve would decrease the resistance provided in the aorta against the left ventricle. The lack of pressure would cause an insufficient closing of the valve and decrease the ability of the ventricle to reach the full volume and internal pressure required to efficiently pump blood up and out of the heart.

Total spinal anesthesia reduces blood pressure because: A. The arterial baroreceptor afferents are not functioning B. Vasopressin secretion falls to zero C. Parasympathetic nerve activity to the heart is eliminated D. Sympathetic nerve activity to the vasculature and heart is eliminated E. Activity in the arterial baroreceptor afferents is reduced

- answer is D Total spinal anesthesia blocks the SNS affects to the heart and vasculature leading to reduced vascular tone and blood pressure

During the cardiac cycle, which of the following occurs when left ventricular pressure rises above aortic pressure? A. The mitral valve closes B. The aortic valve closes C. The mitral valve opens D. The aortic valve opens E. The pulmonary valve closes.

- answer is D What occurs when left ventricular pressure rises above aortic pressure? This causes the aortic valve to open. The mitral valve closes when ventricular pressure rises above atrial pressure. The aortic valve closes when ventricular pressure decreases below aortic pressure. The mitral valve opens when ventricular pressure falls below atrial pressure. The pulmonary valve closes when right ventricular pressure falls pulmonary arterial pressure.

At rest, VO2 is within the rang of _______________ for a healthy individual A. 1500 - 2000 ml O2/(min*kg) B. 15 - 20 ml O2/(min*kg) C. 150 - 200 ml O2/(min*kg) D. 1.5 - 2 ml O2/(min*kg)

- answer is D. 1.5 - 2 mL O2/(min*Kg)

During the cross-bridge cycle, hydrolysis of ATP leads to: A. binding of myosin head to actin filament B. release of ADP C. dissociation of myosin head from actin filament D. activation of myosin head E. the "power stroke"

- answer is D. Hydrolysis of ATP leads to activation of the myosin head. The energy from hydrolyzing ATP to ADP+Pi cocks the myosin head back into the active position.

LO represents A. Shortest sarcomere length B. Longest sarcomere length C. Sub-optimal sarcomere overlap D. Optimal sarcomere overlap E. None of the above

- answer is D. Lo represents optimal sarcomere overlap. Again, this is just recall, but please understand the different points of length-tension curve

What is the most physiologically relevant marker of performance? A. Work B. Load (Tension) C. Velocity D. Power

- answer is D. Power

Slow Twitch muscle fibers A. have characteristically high myoglobin levels B. are fatigue resistant C. have few mitochondria D. have characteristically high myoglobin levels and are fatigue resistant E. have characteristically high myoglobin levels and have few mitochondria

- answer is D. Slow Twitch muscle fibers are recruited quickly and fatigue last. They have lots of mitochondria and myoglobin to support the energy needs.

Which of the following vessels has the highest resistance? A. capillaries B. venuoles C. veins D. arterioles E. aorta

- answer is D. arterioles

Exercise training leads to a greater VO2max by increasing _______________ A. All of these B. resting HR C. capillary density D. capillary density and mitochondria number E. mitochondria number

- answer is D. capillary density and mitochondria number

Which of the following will tend to increase venous return? A. increased right atrial pressure B. hemorrhage C. breathing out D. increased ventricular contractility E. standing

- answer is D. increased ventricular contractility

The motor neuron releases this neurotransmitter into the post-synaptic cleft of the neuromuscular junction to invoke an end-plate potential. A. Calcium B. Nicotine C. Potassium D. Acetylcholine

- answer is D: acetylcholine 1) end-plate potential initiation: by Ach rising phase: simultaneous increase in Na & K falling phase: passive decline in permeabilities due to acetylcholinesterase action potential change: does not exceed -10 mV additional: no regenerative action and no refractory 2) Action Potential initiation: by depolarization rising phase: selective increase Na+ permeability falling phase: selective increase in K+ permeability potential change: reverses polarity additional: regenerative ascent followed by refractory

Aging leads to stiffening of the blood vessels. This leads to increased blood pressure and A. Preload B. Stroke Volume C. Cardiac Output D. Heart Rate E. Afterload

- answer is E

A patient was born with a congential heart defect called "Coarctation of the Aorta," in which the aorta narrows at specific point in the arch, followed by an immediate return to normal diameter size. (Think of it as if a rubber band is restricting the middle of the aortic arch) What would this restriction point int he aortic arch do to blood flow after the coarctation? A. Turbulent Flow B. Laminar Flow C. Increased Flow D. Decreased Velocity E. Increased Pressure

- answer is E (radius ↓: ↑ resistance, ↓ flow) (The left ventricle has to work harder to force blood through the narrowed part of the aorta )

If a depolarizing electrical stimulus was applied to a ventricular myocyte, which of the following would be most likely to occur if extracellular calcium was low (below normal)? A. the magnitude of Phase 0 of the action potential would increase B. Phase 0 of the action potential would be eliminated C. repolarization of the membrane would not occur, i.e., resting membrane potential would not be re-established D. the duration of the plateau phase (Phase 2) of the action potential would increase E. the duration of the plateau phase (Phase 2) of the action potential would decrease

- answer is E If a depolarizing electrical stimulus was applied to a ventricular myocyte, which of the following would be most likely to occur if extracellular calcium was low (below normal)? Cardiac muscle cells rely on extracellular calcium for contraction. The L-type Ca+ channel opens at -40mV and stays open during the plateau phase. If there is less Ca+ available this phase would shorten.

If two successive QRS complexes are 0.6 seconds apart: A. Heart rate is 60 bpm. B. Heart rate is 70 bpm. C. Heart rate is 80 bpm. D. Heart rate is 90 bpm E. Heart rate is 100 bpm.

- answer is E R-R interval is 0.6seconds, HR = 0.6sec/beat *1/60sec/min = 100beats/min

Nitric oxide: A. is generated by endothelial cells B. Is freely permeable to cell membranes C. activates cyclic GMP in vascular smooth muscle cells D. is a potent vasodilator E. all of the above

- answer is E These are all the things NO is known for. You should understand how NO works and how it might be used to treat diseases like neonatal persistant pulmonary hypertension.

What channel maintains transmembrane potential in ventricular cardiomyocytes? A. Voltage Gated Na+ channel B. Delayed Rectifier K+ channel C. T-type Ca++ Channel D. L-type Ca++ Channel E. Inward Rectifier K+ channel

- answer is E - this is the fast response (ventricular/atrial action). 0-4 phase so 5 total - A is wrong - this is phase 0 depolarization - B and D are wrong - this is phase 2, plateua phase. Opening of voltage dependent slow L-type Ca++ channels balance with slow delayed rectifier K+ channels. Influx Ca++, Efflux K+ - C is wrong - no role in fast response

A patient has suffered severe burn injuries to his body. although he has been treated for his burns, and has been hospitalized for several days, the swelling is not under control and he is presenting with signs of dehydration. the most likely explanation for this is A. overheating due to burn injury B. shock due to blood loss C. heart failure D. decreased venous return E. increasing protein permeability in capillaries

- answer is E.

If a person's stroke volume was 70ml, and the end diastolic volume is increased from 135ml to 165ml, without any changes in arterial pressure. What is the stroke volume in the next few cardiac cycles? A. 135 ml B. 165 ml C. 70 ml D. 40 ml E. 100 ml

- answer is E. !00 ml

If you stimulate the neurons in nucleus of the solitary tract that receive input from the arterial baroreceptors you would expect to observe: A. An increase in HR and a decrease in BP B. An increase in HR and increase in BP C. A decrease in HR and no change in BP D. A decrease in HR and an increase in BP E. A decrease in HR and a decrease in BP

- answer is E. A decrease in HR and a decrease in BP

Increase of which molecule will lead to vasodilation? A. Glucose B. Angiotensin II C. Norepinephrine D. O2 E. CO2

- answer is E. CO2

Increase of which molecule will lead to vasodilation? A. Glucose B. Angiotensin II C. Norepinephrine D. O2 E. CO2

- answer is E. CO2

This ion channel opens at a voltage of about -40mV? A. Na+ channel B. Delayed rectifer K+ channel IKr C. T-type Ca++ channel D. Na+/K+ mixed conductance "funny" channel E. L-type Ca++ channel

- answer is E: L-type Ca++ channel - C is wrong: T-type open at -55mV and inactive fairly rapidly

What is the baroreflex and how is it regulated? What does it mean that they are "mechanoreceptors?"

- are mechanoreceptors located in the carotid sinus and in the aortic arch. - sense pressure changes by responding to change in the tension of the arterial wall. - a fast response to changes in blood pressure.

Describe the flow of blood in the fetus.

- blood flow is quite high because the content of oxygen in the blood is relatively low. So to get enough oxygen to the tissues, they need to have high blood flow to get that oxygen there - the left and right side pump in parallel because only small percentage of blood goes to the lungs - the blood in the pulmonary artery, only some of that goes into the lungs and most of it goes through a shunt called the arterial duct (or ductus arteriosis) which takes blood from the pulmonary artery into the arch of the aorta and dumps the blood into the systemic circulation - lungs are not inflated with air, so there is no way for oxygen to get into the fetal circulation through the lungs. - In pregnancy there is a placenta which is a tissue which gets blood supply from both the fetus and the mom - Fetus has umbilical placental circulation, which receives blood supply from fetus and mother. This oxygen coming in thorough the maternal blood ends up in the fetal blood. This arterial comes back into vena cava through venous duct (ductus veniosus), which allows blood to come back into vena cava. - This blood is going to cross the from right atrium to left atrium through a 3rd shunt in the fetal circulation called the foramen ovale.

During rapid ventricular filling phase, venous pressure _________ and aortic pressure _____________

- decreases, decreases

A patient enters the clinic with a systolic pressure of 140 and a diastolic pressure of 80. What is MAP? A. 100 B. 60 C. 80 D. 120 E. 160

100

Given the following parts of the autonomic reflex, select the correct order from input stimulus to the resulting action for an autonomic reflex 1. afferent neuron 2. effector cell 3. preganglionic neuron 4. sensory receptor 5. central nervous system Select the correct order from input stimulus to the resulting action for an autonomic reflex:

4,1,5,3,2 4. sensory receptor 1. afferent neuron 5. central nervous system 3. preganglionic neuron 2. effector cell

In an individual, the following values are measured: site & mean pressure (mm Hg) Aorta 96 mm Hg Right Atrium 3 mm Hg Pulmonary Artery 20 mm Hg Left Atrium 8 mm Hg cardiac output in this patient is 6 L/min. A) Calculate the pulmonary vascular resistance. B) Calculate the systemic vascular resistance.

A) 2 mmHg/L/min B) 15/5 mmHg/L/min

In a contracting muscle the Z lines come closer together. A) True B) False

A) True

The sequence of cross bridge formation and myofilament movement will be repeated as long as calcium ions are present. A) True B) False

A) True

When cross bridges form and the muscle fibers contract, the actin myofilament slides past the myosin myofilament. A) True B) False

A) True

The bond between the actin and myosin head is broken when... A) an ATP molecule binds to the myosin head. B) an ATP molecule binds to the actin molecule. C) an ATP molecule breaks down on the myosin head. D) an ATP molecule breaks down on the actin molecule. E) ADP and phosphate bind to the myosin head.

A) an ATP molecule binds to the myosin head

The primary neurotransmitter at the neuromuscular junction is A) dopamine. B) adrenaline. C) acetylcholine. D) histamine. E) serotonin.

C) acetylcholine

An action potential arriving at the presynaptic terminal causes... A) voltage-gated sodium ion channels to open, and sodium ions to diffuse into the cell. B) voltage-gated sodium ion channels to open, and sodium ions to diffuse out of the cell. C) voltage-gated calcium ion channels to open, and calcium ions to diffuse into the cell. D) acetylcholine to diffuse into the cell. E) ligand-gated sodium channels to open, and sodium ions to diffuse out of the cell. [Function of the Neuromuscular Junction]

C) voltage-gated calcium ion channels to open, and calcium ions to diffuse into the cell

Which protein in smooth muscle has a similar function as troponin/tropomyosin do in skeletal muscle? A.) Actin B.) Rho kinase C.) Myosin phosphatase D.) Calmodulin

D.) Calmodulin

Which of the following is the order of the vasculature in series starting with left heart > aorta and ending with right atrium

Left Heart > Aorta > Arteries > Arterioles > Capillaries > Venuoles > Veins > Vena Cava > Right Atrium

Which of the following enzymes is used for cross-bridge cycling? A.) Actin phosphatase B.) Tropomyosin phosphorylase C.) Myosin phosphatase D.) Myosin phosphorylase E.) Actin phosphorylase

Myosin phosphatase

A patient has come to the ER with severe dehydration. Her heart rate is 110bpm, her BP is 80/50, and she is non-responsive (unconscious). Your first response is to give her a bolus of saline intravenously. How is her cardiac output affected? Describe the Frank-Starling Law, and how it relates to her care.

The Frank Starling law states that the myocyte length is proportional to the force of contractio, that the increased venous return leads to an increased in EDV and that the heart will ejec the same amount that is pumped into it. If a patient presents with low BP due to dehydration their overall blood volume is low, so the amount of blood being pumped out is lower than it should be. When adding a saline bolus the volume of circulating fluids would increase, thus increasing the volume of blood pumped into and out of the heart. Increasing the preload in the heart would because by an increase in the blood volume, increasing the preload increases the CO and therefore increases overall BP of the PT.

In the isovolumetric ventricular contraction phase of the cardiac cycle the mitral valve is closed and the aortic valve is ________________. A. opened B. closed

closed

In the isovolumetric ventricular relaxation phase of the cardiac cycle the aortic valve is closed and the mitral valve is ______________. A. opened B. closed

closed

The aortic valve opens when the ventricular pressure is __________ the aortic pressure. A. less than B. greater than

greater than

Understand pulmonary vascular resistance - What effect does recruitment have? - What effect does distension have? - What effect does altitude have? - What are some ways to increase pulmonary vascular resistance? - What are some ways to decrease pulmonary vascular resistance?

resistance = change in pressure/ flow (Q) 1) recruitment and distension both contribute to the fact that as pulmonary arterial pressure increase (so when BP increases), pulmonary vascular resistance decreases - An increase in BP induces both recruitment and then distension. * ex. increases CO with exercise - recruitment - as pulmonary arterial pressure increases, blood flows through capillaries that previously had little or no blood flow. This happens especially in the apex (or top) of the lung - distension - when capillaries are already open, further increase in BP dilate the vessels. Increased diameter for blood to flow through. A pressure increase can simultaneously cause recruitment and distension, but recruitment usually happens first. Pathways increase in radius lead to decrease in pulmonary vascular resistance with small increase in pulmonary arterial pressure. - small increases in pulmonary arterial pressure cause recruitment then distension of pulmonary arterial capillaries 2) High altitude causes - generalized vasoconstriction increases pulmonary arterial pressure 3) pulmonary vascular resistance increased by: 4) pulmonary vascular resistance decreased by: - exercise, cardiac output and pulmonary blood flow increases while pulmonary vascular resistance decreases - as pulmonary arterial pressure increases


Set pelajaran terkait

Exam 4 - Chapter 24: Management of Patients With Chronic Pulmonary Disease

View Set

Carbon Chemistry (8th Grade Chemistry: Chapter 4)

View Set

Compound and Complex Sentences Assignments

View Set

SOLVING LOGARITHMIC EQUATIONS USING TECHNOLOGY

View Set

Introduction to Microcomputing - Ch8 - Excel

View Set